Espinor de Dirac y espinor de Weyl

¿Cómo se puede demostrar que el espinor de Dirac es la suma directa de un espinor de Weyl de mano derecha y un espinor de Weyl de mano izquierda?

EDITAR: - Dejar ψ L y ψ R Ser espinores de Weyl zurdos y diestros de 2 componentes. Sus propiedades de transformación son conocidas. Cuando pongo estos dos espinores en una columna y construyo una columna de cuatro componentes que es una suma directa de ψ L y ψ R es decir, ψ D = ψ L ψ R . Esto lo definí como el espinor de Dirac. ¿Bien? Dado que es una suma directa bajo la transformación de Lorentz, la matriz de transformación de Lorentz correspondiente es diagonal. ¿Bien? Entonces es fácil demostrar que satisface la ecuación de Dirac en base quiral. ¿Bien? Esto es posible porque partimos de la definición de espinores de Weyl zurdos y diestros y se conocen sus propiedades de transformación. ¿Bien? Esto se lleva a cabo explícitamente en el libro de Lewis Ryder. Pero supongamos que empiezo al revés. Resuelvo la ecuación de Dirac en base quiral. Entonces nadie me dice que los dos componentes superiores son realmente para zurdos y los dos inferiores son realmente para diestros. Supongamos que tomo esta solución de base quiral de la ecuación de Dirac y ahora la tomo como mi definiciónde Dirac spinor. Entonces, ¿cómo puedo mostrar lo contrario, que está formado por dos irreps del grupo de Lorentz, es decir, ψ D = ψ L ψ R ?

Desde el punto de vista de la teoría de la representación, generalmente se toma como la definición de la representación del espinor de Dirac. ¿Con qué definición estás trabajando si no es por esa?
Estoy tomando la definición de espinor de Dirac como la solución de la ecuación de dirac, no la definición que se usa en la teoría de la representación del grupo de Lorentz.
Entonces la pregunta correcta sería: ¿Por qué los espinores de Dirac son, salvo la acción de una matriz no singular que preserva las relaciones de conmutación de gamma marices , la suma directa de los dos tipos de espinores de Weyl?
Lo siento, por definición en las transformaciones de Lorentz de base Chiral/Weyl es "diagonal" (es decir, no mezcla los dos componentes) porque está hecho de dos representaciones separadas. Entonces no puedo entender tu pregunta.
@Roopam Sinha Gracias, ahora entiendo. Por favor, te sugiero que no uses mayúsculas, parece que estás gritando y no suena muy educado.
Trate de usar algo o algo en su lugar para enfatizar.
¡Ahora se ve mejor!

Respuestas (3)

A partir de la covarianza relativista de la ecuación de Dirac (consulte la Sección 2.1.3 en el libro QFT de Itzykson y Zuber para obtener una derivación. También sigo más o menos su notación), sabe cómo se transforma un espinor de Dirac. Uno tiene

ψ ( X ) = S ( Λ )   ψ ( X )
bajo la transformación de Lorentz
X m = Λ m v   X v = Exp ( λ ) m v   X v = ( I + λ m v + )   X v   .
Explícitamente, uno tiene S ( Λ ) = Exp ( 1 8 [ γ m , γ v ]   λ m v ) .

Para mostrar la reducibilidad, todo lo que necesita es encontrar una base para las matrices gamma (así como los espinores de Dirac) tal que [ γ m , γ v ] es un bloque diagonal con dos 2 × 2 bloques Una vez que se muestra esto, se prueba la reducibilidad de los espinores de Dirac bajo transformaciones de Lorentz ya que S ( Λ ) también es bloque-diagonal. Tal base se llama base quiral. También es importante tener en cuenta que un término de masa en el término de Dirac mezcla los espinores de Weyl en la ecuación de Dirac, pero eso no es un problema para la reducibilidad.

Si bien esta derivación no usa directamente la teoría de representación del grupo de Lorentz, sí usa la covarianza de Lorentz de la ecuación de Dirac. No sé si esto es lo que querías.

( No estoy interesado en su generosidad, por favor no me otorgue nada ) .

La respuesta se reduce a la teoría de la representación del grupo de Lorentz. Se puede encontrar una buena discusión en el primer volumen de QFT de Weinberg (y también en otros lugares). Una cosa a tener en cuenta es que postulas una representación de 4 dimensiones del grupo de Lorentz. Este postulado surge cuando asumes que tus objetos tienen 4 componentes. Ahora, una representación de 4 dimensiones del grupo de Lorentz puede ser irreducible, correspondiente a los 4 vectores, o estar construida por dos representaciones bidimensionales, correspondientes a dos espinores de 2 componentes. Estas son las únicas dos opciones.

No hay nada que pueda discernir la izquierda de la derecha. Todo lo que sabemos es que habrá dos subespacios bidimensionales que son independientes entre sí. (Esto se puede ver en la base quiral de las matrices gamma). Simplemente llamamos a los objetos en uno de los espacios que se mueven a la izquierda y a los objetos en el otro espacio que se mueven a la derecha. Sin embargo, los dos espacios son absolutamente idénticos. Por ejemplo, si escribimos la ecuación de Dirac en forma de dos componentes (y hay una forma equivalente de hacer todos los cálculos posibles usando solo espinores de dos componentes en lugar de espinores de 4 componentes [1]), entonces podemos ver que las ecuaciones satisfechos por los espinores izquierdo y derecho son absolutamente equivalentes.

¡Espero que esto ayude!

[1] http://arxiv.org/abs/0812.1594

El punto clave al escribir una acción para espinores es la existencia de un álgebra de Clifford (expandida por las matrices gamma )

{ γ a , γ b } = 2 η a b 1 ,
donde el indice a corre de 0 a 3 (o 0 a D 1 , dónde D es la dimensión del espacio-tiempo).

Toda la base para el álgebra está dada por el γ 's y todos los productos posibles... debido a la última ecuación, solo contribuyen los productos antisimétricos.

Es posible demostrar que para todo espaciotiempo de dimensión par, el producto (antisimétrico) de todos γ 's, es decir , γ γ 0 γ D 1 permite definir proyectores no triviales

PAG + 2 = PAG + , PAG 2 = PAG , PAG + PAG = 0.

Estos proyectores sirven para dividir el espinor en pedazos,

ψ ± = PAG ± Ψ ,
con Ψ el espinor de Dirac, y ψ ± los de Weyl (o quirales).

Conclusión

El hecho de que los espinores de Dirac se puedan dividir en los de Weyl se debe a la dimensionalidad del espacio-tiempo. En dimensión impar, los proyectores son triviales porque son 0 y 1 respectivamente.


Comentario adicional: los espinores de Weyl son bidimensionales solo si se utiliza la representación quiral de matrices gamma. De lo contrario, tienen la mitad de los componentes (en términos de dimensión real de los espinores).